Differentiating :calculus theory

Click For Summary
The discussion centers on verifying the identity arctan(x) + arctan(1/x) = π/2 using calculus theory. Participants suggest differentiating the left-hand side, leading to the conclusion that the derivative f'(x) = 1/(1+x²) - 1/(x²+1) equals zero for all x ≠ 0, indicating the function is constant. This implies that the identity holds true across its domain, with a note that x should be greater than zero for clarity. The final step involves substituting a specific value of x to determine the constant. The verification process confirms the identity as correct.
chapsticks
Messages
38
Reaction score
0

Homework Statement




Verify the identity:

arctanx + arctan(1/x)=∏/2

using calculus theory.
(Hint: Differentiate the left hand side of the identity)

Homework Equations



?

The Attempt at a Solution


is this correct?

tan(arctanx + arctan(1/x))
= [tan(arctan(x)) + tan(arctan(1/x))][1 - tan(arctan(x))*tan(arctan(1/x))]

= [x + 1/x]/[1 - x*1/x] = (x + 1/x)/0 = oo

tan pi/2 = oo
 
Physics news on Phys.org
mmm... they wanted you to do this:
\frac{1}{1+x^2}+ etc...
 
where did that come from I'm confused??
 
chapsticks said:

Homework Statement




Verify the identity:

arctanx + arctan(1/x)=∏/2

using calculus theory.
(Hint: Differentiate the left hand side of the identity)

Homework Equations



?

The Attempt at a Solution


is this correct?

tan(arctanx + arctan(1/x))
= [tan(arctan(x)) + tan(arctan(1/x))][1 - tan(arctan(x))*tan(arctan(1/x))]

= [x + 1/x]/[1 - x*1/x] = (x + 1/x)/0 = oo

tan pi/2 = oo

Start by using the hint, which is to differentiate the left side.
 
f'(x)=1/(1+x2) + -1/(x2+1) =0

for all x≠0

function is constant on domain

is this right?
 
chapsticks said:
f'(x)=1/(1+x2) + -1/(x2+1) =0

for all x≠0

function is constant on domain

is this right?

Yes, it is. I guessing they meant to specify x>0. Now just pick a nice value of x to put into figure out what the constant is.
 
Question: A clock's minute hand has length 4 and its hour hand has length 3. What is the distance between the tips at the moment when it is increasing most rapidly?(Putnam Exam Question) Answer: Making assumption that both the hands moves at constant angular velocities, the answer is ## \sqrt{7} .## But don't you think this assumption is somewhat doubtful and wrong?

Similar threads

  • · Replies 40 ·
2
Replies
40
Views
4K
  • · Replies 5 ·
Replies
5
Views
2K
  • · Replies 8 ·
Replies
8
Views
2K
  • · Replies 3 ·
Replies
3
Views
2K
  • · Replies 1 ·
Replies
1
Views
2K
  • · Replies 2 ·
Replies
2
Views
2K
  • · Replies 2 ·
Replies
2
Views
1K
Replies
9
Views
2K
  • · Replies 15 ·
Replies
15
Views
2K
  • · Replies 3 ·
Replies
3
Views
1K